Electronics Repair (sci.electronics.repair) Discussion of repairing electronic equipment. Topics include requests for assistance, where to obtain servicing information and parts, techniques for diagnosis and repair, and annecdotes about success, failures and problems.

Reply
 
LinkBack Thread Tools Search this Thread Display Modes
  #1   Report Post  
Tom MacIntyre
 
Posts: n/a
Default Lightning protection


Hi folks...this was in a baseball newsgroup that I participate in
(rec.sport.baseball). Faraday cages have been mentioned, implying that
it could be safe. I don't know that I'd trust this against lightning
(maybe a Van de Graff generator). What do you think?

"Just wondering if sitting on a wood bench inside a metal cage (i.e.
metal on top and 4 sides) is a safe place to be during a lightning
storm. This is similar to the situation of being in a car (except for
the rubber tires!). I have seen younger kids wait out a passing storm
this way and am wondering if it IS in fact safe? What would the effect
of metal cleats be? I'd appreciate any thoughts from anybody but
especially from physics teachers/physicists. Thanks!"

Tom
  #2   Report Post  
TimPerry
 
Posts: n/a
Default


"Tom MacIntyre" wrote in message
...

Hi folks...this was in a baseball newsgroup that I participate in
(rec.sport.baseball). Faraday cages have been mentioned, implying that
it could be safe. I don't know that I'd trust this against lightning
(maybe a Van de Graff generator). What do you think?

"Just wondering if sitting on a wood bench inside a metal cage (i.e.
metal on top and 4 sides) is a safe place to be during a lightning
storm. This is similar to the situation of being in a car (except for
the rubber tires!). I have seen younger kids wait out a passing storm
this way and am wondering if it IS in fact safe? What would the effect
of metal cleats be? I'd appreciate any thoughts from anybody but
especially from physics teachers/physicists. Thanks!"

Tom


there is no guarantee that a baseball cage is adequately grounded. the
support poles are usually set in concrete.

faraday cages are meant to block RF. lightning is DC (mostly).

refer to http://www.lightningsafety.com/nlsi_pls/lst.html


http://www.securityworld.com/communi...ingsafety.html

http://www.nhoem.state.nh.us/mitigation/fig%203-16.htm

http://www.metlife.com/Applications/...,P4091,00.html

http://www.scouter.com/compass/Leade...htning_Safety/


i routinely repair lightning damaged equipment. when you see first hand
large molten globs of metal, ceramic, and other material that used to be
perfectly good electronics you tend to get real cautious about where you go
and what you do when a lightning storm passes through.







  #3   Report Post  
w_tom
 
Posts: n/a
Default

Effectiveness of protection from lightning is determined by how that
electricity gets to earth ground. Not earth ground beneath you. If
seeks charges via earth that is maybe miles away. In the process, the
struck batting cage then carries electricity underneath your feet to to
take a mile plus path to those distant charges.

For example, if standing on earth with feet apart, and if the nearby
tree is struck, then electricity might pass through earth, up one leg,
down the other, and then continue through earth. Therefore the human
is electrocuted.

This is also why four legged animals tend to be at risk when a nearby
tree is struck. Four legged animals do not make a single point ground
connection to earth; therefore can be harmed by electricity flowing
through the earth.

The baseball cage may act as a good lightning rod. But then
electricity passes beneath a human to cause harm. IOW the faraday cage
is not complete because the earth beneath the human was not
sufficiently conductive.

BTW, lightning is not DC. Lightning is AC current - mostly in radio
frequency ranges. This AC nature is why longer conductors, sufficient
for 60 Hz AC electric power, can be poor conductors to lightning; its
energy in higher frequencies. This AC nature of lightning is why so
many humans think lightning is capricious.

Tom MacIntyre wrote:
Hi folks...this was in a baseball newsgroup that I participate
in (rec.sport.baseball). Faraday cages have been
mentioned, implying that it could be safe. I don't
know that I'd trust this against lightning

(maybe
a Van de Graff generator). What do you think?


  #4   Report Post  
w_tom
 
Posts: n/a
Default

Effectiveness of protection from lightning is determined by how that
electricity gets to earth ground. Not earth ground beneath you. If
seeks charges via earth that is maybe miles away. In the process, the
struck batting cage then carries electricity underneath your feet to to
take a mile plus path to those distant charges.

For example, if standing on earth with feet apart, and if the nearby
tree is struck, then electricity might pass through earth, up one leg,
down the other, and then continue through earth. Therefore the human
is electrocuted.

This is also why four legged animals tend to be at risk when a nearby
tree is struck. Four legged animals do not make a single point ground
connection to earth; therefore can be harmed by electricity flowing
through the earth.

The baseball cage may act as a good lightning rod. But then
electricity passes beneath a human to cause harm. IOW the faraday cage
is not complete because the earth beneath the human was not
sufficiently conductive.

BTW, lightning is not DC. Lightning is AC current - mostly in radio
frequency ranges. This AC nature is why longer conductors, sufficient
for 60 Hz AC electric power, can be poor conductors to lightning; its
energy in higher frequencies. This AC nature of lightning is why so
many humans think lightning is capricious.

Tom MacIntyre wrote:
Hi folks...this was in a baseball newsgroup that I participate
in (rec.sport.baseball). Faraday cages have been
mentioned, implying that it could be safe. I don't
know that I'd trust this against lightning

(maybe
a Van de Graff generator). What do you think?


  #5   Report Post  
NSM
 
Posts: n/a
Default


"TimPerry" wrote in message
...

i routinely repair lightning damaged equipment. when you see first hand
large molten globs of metal, ceramic, and other material that used to be
perfectly good electronics you tend to get real cautious about where you

go
and what you do when a lightning storm passes through.


Yep, you never know. I hide under the bed myself.

N




  #6   Report Post  
TimPerry
 
Posts: n/a
Default

BTW, lightning is not DC. Lightning is AC current - mostly in radio
frequency ranges. This AC nature is why longer conductors, sufficient
for 60 Hz AC electric power, can be poor conductors to lightning; its
energy in higher frequencies. This AC nature of lightning is why so
many humans think lightning is capricious.


utter and complete crap.

lightning is an electrostatic discharge.



  #7   Report Post  
N Cook
 
Posts: n/a
Default

"w_tom" wrote in message
oups.com...
Effectiveness of protection from lightning is determined by how that
electricity gets to earth ground. Not earth ground beneath you. If
seeks charges via earth that is maybe miles away. In the process, the
struck batting cage then carries electricity underneath your feet to to
take a mile plus path to those distant charges.

For example, if standing on earth with feet apart, and if the nearby
tree is struck, then electricity might pass through earth, up one leg,
down the other, and then continue through earth. Therefore the human
is electrocuted.

This is also why four legged animals tend to be at risk when a nearby
tree is struck. Four legged animals do not make a single point ground
connection to earth; therefore can be harmed by electricity flowing
through the earth.

The baseball cage may act as a good lightning rod. But then
electricity passes beneath a human to cause harm. IOW the faraday cage
is not complete because the earth beneath the human was not
sufficiently conductive.

BTW, lightning is not DC. Lightning is AC current - mostly in radio
frequency ranges. This AC nature is why longer conductors, sufficient
for 60 Hz AC electric power, can be poor conductors to lightning; its
energy in higher frequencies. This AC nature of lightning is why so
many humans think lightning is capricious.

Tom MacIntyre wrote:
Hi folks...this was in a baseball newsgroup that I participate
in (rec.sport.baseball). Faraday cages have been
mentioned, implying that it could be safe. I don't
know that I'd trust this against lightning

(maybe
a Van de Graff generator). What do you think?



Lightning is AC ?!!!. I've never seen any rotating clouds,
in the UK we get plenty of lighning but no rotating tornados with lightning.

--
Diverse Devices, Southampton, England
electronic hints and repair briefs , schematics/manuals list on
http://home.graffiti.net/diverse:graffiti.net/





  #8   Report Post  
Choreboy
 
Posts: n/a
Default

w_tom wrote:


The baseball cage may act as a good lightning rod. But then
electricity passes beneath a human to cause harm. IOW the faraday cage
is not complete because the earth beneath the human was not
sufficiently conductive.


With no mesh underfoot, it might depend on the shape of the cage. If
you were surrounded on three sides, there might not be much current
underfoot because the metal fence would be a better conductor. At the
nearest playground, the "batter's cage" is just a wide V backstop, so I
wouldn't stand there in a storm.

Pointy rods on the top of the cage might prevent a lot of strikes by
bleeding off charges.

BTW, lightning is not DC. Lightning is AC current - mostly in radio
frequency ranges. This AC nature is why longer conductors, sufficient
for 60 Hz AC electric power, can be poor conductors to lightning; its
energy in higher frequencies. This AC nature of lightning is why so
many humans think lightning is capricious.

I think I've read that there can be several phases in a lightning
strike, and maybe electricity flows back and forth. I can't remember.
Anyway, lightning doesn't have to be AC for a power line to be a poor
conductor. Inductance reacts against a sudden change in current. With
lightning, the change is so sudden that a little inductance means a lot
of impedance.

Leading from a 220V transformer to a house, the three conductors are
usually twisted together nowadays. The reduces inductance. I'll bet
we'd have less damage from surges with fast rise times, whether or not
caused by lightning, if power companies spaced the conductors a foot
apart as they once did.
  #9   Report Post  
CJT
 
Posts: n/a
Default

w_tom wrote:
snip
BTW, lightning is not DC. Lightning is AC current - mostly in radio
frequency ranges. This AC nature is why longer conductors, sufficient
for 60 Hz AC electric power, can be poor conductors to lightning; its
energy in higher frequencies. This AC nature of lightning is why so
many humans think lightning is capricious.


Huh? Cite?


snip
-
The e-mail address in our reply-to line is reversed in an attempt to
minimize spam. Our true address is of the form .
  #10   Report Post  
w_tom
 
Posts: n/a
Default

To make it easier for some to understand, I should have first noted
that lightning is an impulse. The classic current transient is an 8/20
microsecond impulse. That impulse is a summation of frequency
components; AC waveforms at various radio frequencies. This is basic
knowledge to those with electrical engineering training AND those with
math backgrounds that include Fourier Series.

Lightning obviously is not DC. If it was, then only wire resistance
would be relevant. An impulse due to lightning contains numeorus AC
sine waves. Much of that energy is in the 100 kilohertz and Megahertz
range. These obviously are not DC. Wire resistance has very little
relevance. Wire impedance is a dominant parameter due to sine waves in
an 8/20 usec impulse called CG lightning. Wire impedance would be
completely irrelevant if lightning were DC.

Nothing new about this. First year electrical engineers even study
impulse response. Those who have a problem with this concept will have
difficulty understanding the electrical nature of lightning. Clearly
lightning is not DC because lightning current is an impulse in the
microsecond range.
CJT wrote:
w_tom wrote:
BTW, lightning is not DC. Lightning is AC current - mostly in radio
frequency ranges. This AC nature is why longer conductors, sufficient
for 60 Hz AC electric power, can be poor conductors to lightning; its
energy in higher frequencies. This AC nature of lightning is why so
many humans think lightning is capricious.


Huh? Cite?




  #11   Report Post  
Aidan Grey
 
Posts: n/a
Default

To all:

This statement is correct. A lightning bolt often consists of as many as
40 to 50 individual discharges. This is not apparent unless the lightning
is captured by high speed photography.

So, since the lightning discharge turns on and off 40 to 50 times in a
fraction
of a second, it is not pure DC.

The current flows in only one direction. However, the energy is split
between a DC component, and a considerable part that is effectively RF
energy. Note this is RF energy, not low frequency AC.

I don't have a Web source to cite for this, but I have read about this
before. Specifically, that a ground for a lightning rod must not go through
a coil of wire.


Aidan Grey


On 5 Jul 2005 06:40:58 -0700, w_tom wrote:

Effectiveness of protection from lightning is determined by how that
electricity gets to earth ground. Not earth ground beneath you. If
seeks charges via earth that is maybe miles away. In the process, the
struck batting cage then carries electricity underneath your feet to to
take a mile plus path to those distant charges.

For example, if standing on earth with feet apart, and if the nearby
tree is struck, then electricity might pass through earth, up one leg,
down the other, and then continue through earth. Therefore the human
is electrocuted.

This is also why four legged animals tend to be at risk when a nearby
tree is struck. Four legged animals do not make a single point ground
connection to earth; therefore can be harmed by electricity flowing
through the earth.

The baseball cage may act as a good lightning rod. But then
electricity passes beneath a human to cause harm. IOW the faraday cage
is not complete because the earth beneath the human was not
sufficiently conductive.

BTW, lightning is not DC. Lightning is AC current - mostly in radio
frequency ranges. This AC nature is why longer conductors, sufficient
for 60 Hz AC electric power, can be poor conductors to lightning; its
energy in higher frequencies. This AC nature of lightning is why so
many humans think lightning is capricious.

Tom MacIntyre wrote:
Hi folks...this was in a baseball newsgroup that I participate
in (rec.sport.baseball). Faraday cages have been
mentioned, implying that it could be safe. I don't
know that I'd trust this against lightning

(maybe
a Van de Graff generator). What do you think?





  #12   Report Post  
N Cook
 
Posts: n/a
Default

"Aidan Grey" wrote in message
k.com...
To all:

This statement is correct. A lightning bolt often consists of as many

as
40 to 50 individual discharges. This is not apparent unless the lightning
is captured by high speed photography.

So, since the lightning discharge turns on and off 40 to 50 times in a
fraction
of a second, it is not pure DC.

The current flows in only one direction. However, the energy is split
between a DC component, and a considerable part that is effectively RF
energy. Note this is RF energy, not low frequency AC.

I don't have a Web source to cite for this, but I have read about this
before. Specifically, that a ground for a lightning rod must not go

through
a coil of wire.


Aidan Grey


On 5 Jul 2005 06:40:58 -0700, w_tom wrote:

Effectiveness of protection from lightning is determined by how that
electricity gets to earth ground. Not earth ground beneath you. If
seeks charges via earth that is maybe miles away. In the process, the
struck batting cage then carries electricity underneath your feet to to
take a mile plus path to those distant charges.

For example, if standing on earth with feet apart, and if the nearby
tree is struck, then electricity might pass through earth, up one leg,
down the other, and then continue through earth. Therefore the human
is electrocuted.

This is also why four legged animals tend to be at risk when a nearby
tree is struck. Four legged animals do not make a single point ground
connection to earth; therefore can be harmed by electricity flowing
through the earth.

The baseball cage may act as a good lightning rod. But then
electricity passes beneath a human to cause harm. IOW the faraday cage
is not complete because the earth beneath the human was not
sufficiently conductive.

BTW, lightning is not DC. Lightning is AC current - mostly in radio
frequency ranges. This AC nature is why longer conductors, sufficient
for 60 Hz AC electric power, can be poor conductors to lightning; its
energy in higher frequencies. This AC nature of lightning is why so
many humans think lightning is capricious.

Tom MacIntyre wrote:
Hi folks...this was in a baseball newsgroup that I participate
in (rec.sport.baseball). Faraday cages have been
mentioned, implying that it could be safe. I don't
know that I'd trust this against lightning

(maybe
a Van de Graff generator). What do you think?






chopped DC makes sense - anyone know what initiates the chopping and the
rate of chop?
Is chopped DC the discharge mechanism from van de graff's etc?


  #13   Report Post  
CJT
 
Posts: n/a
Default

Aidan Grey wrote:
To all:

This statement is correct. A lightning bolt often consists of as many as
40 to 50 individual discharges. This is not apparent unless the lightning
is captured by high speed photography.

So, since the lightning discharge turns on and off 40 to 50 times in a
fraction
of a second, it is not pure DC.

The current flows in only one direction. However, the energy is split
between a DC component, and a considerable part that is effectively RF
energy. Note this is RF energy, not low frequency AC.


A small multiple of 40-50 Hz is not exactly RF. So what's the risetime
of these pulses? Duration?


I don't have a Web source to cite for this, but I have read about this
before. Specifically, that a ground for a lightning rod must not go through
a coil of wire.


Why would you want one to run through a coil, anyway?

I suspect the warning is to avoid somebody being electrocuted by induced
current from the pulse(s).



Aidan Grey


On 5 Jul 2005 06:40:58 -0700, w_tom wrote:

Effectiveness of protection from lightning is determined by how that
electricity gets to earth ground. Not earth ground beneath you. If
seeks charges via earth that is maybe miles away. In the process, the
struck batting cage then carries electricity underneath your feet to to
take a mile plus path to those distant charges.

For example, if standing on earth with feet apart, and if the nearby
tree is struck, then electricity might pass through earth, up one leg,
down the other, and then continue through earth. Therefore the human
is electrocuted.

This is also why four legged animals tend to be at risk when a nearby
tree is struck. Four legged animals do not make a single point ground
connection to earth; therefore can be harmed by electricity flowing
through the earth.

The baseball cage may act as a good lightning rod. But then
electricity passes beneath a human to cause harm. IOW the faraday cage
is not complete because the earth beneath the human was not
sufficiently conductive.

BTW, lightning is not DC. Lightning is AC current - mostly in radio
frequency ranges. This AC nature is why longer conductors, sufficient
for 60 Hz AC electric power, can be poor conductors to lightning; its
energy in higher frequencies. This AC nature of lightning is why so
many humans think lightning is capricious.

Tom MacIntyre wrote:
Hi folks...this was in a baseball newsgroup that I participate
in (rec.sport.baseball). Faraday cages have been
mentioned, implying that it could be safe. I don't
know that I'd trust this against lightning

(maybe
a Van de Graff generator). What do you think?






--
The e-mail address in our reply-to line is reversed in an attempt to
minimize spam. Our true address is of the form .
  #14   Report Post  
w_tom
 
Posts: n/a
Default

Risetime of these current pulses was provided in a previous
post: exponentially rising and falling edges of 8/20
microseconds.

Obviously the ground wire would not be run through a coil.
But due to the high frequency nature of transients, that
ground wire must not be spliced, no sharp bends, and not
routed inside metallic materials. A violation would only
increase wire impedance unacceptably. Why? Because even
sharp bends cause significant impedance increases when
discussing the higher frequency components of lightning.

We could play a game of 50,000 questions since even the most
basic nature of lightning - its RF components - is new
information. These and other questions have been answered
previously in a list of citations at:
http://makeashorterlink.com/?X61C23DCA

Figure about one days worth of reading.

CJT wrote:
A small multiple of 40-50 Hz is not exactly RF. So what's the
risetime of these pulses? Duration?
...

Why would you want one to run through a coil, anyway?

I suspect the warning is to avoid somebody being electrocuted by induced
current from the pulse(s).

  #15   Report Post  
TimPerry
 
Posts: n/a
Default


"w_tom" wrote in message
...
Risetime of these current pulses was provided in a previous
post: exponentially rising and falling edges of 8/20
microseconds.

Obviously the ground wire would not be run through a coil.
But due to the high frequency nature of transients, that
ground wire must not be spliced, no sharp bends, and not
routed inside metallic materials. A violation would only
increase wire impedance unacceptably. Why? Because even
sharp bends cause significant impedance increases when
discussing the higher frequency components of lightning.

We could play a game of 50,000 questions since even the most
basic nature of lightning - its RF components - is new
information. These and other questions have been answered
previously in a list of citations at:
http://makeashorterlink.com/?X61C23DCA



citing a newsgroup discussion is not a valid reference.

elaborate (and probably inaccurate) discussion as to the exact nature of
lightning will not be of benefit to the original poster.

a DC pulse is still a DC pulse no amount of wishful thinking will turn it in
to AC (granted that there are some aspects that generate RF and perhaps some
other characteristics that may be able to be handled by AC analysis)

looking at a few of toms 16,900 archived posts i see he also claims that a
ground rod in rock will make a perfectly good ground. from that it follows
that a batting cage set in concrete is adiquitly grounded. i dont think so.






Figure about one days worth of reading.

CJT wrote:
A small multiple of 40-50 Hz is not exactly RF. So what's the
risetime of these pulses? Duration?
...

Why would you want one to run through a coil, anyway?

I suspect the warning is to avoid somebody being electrocuted by induced
current from the pulse(s).





  #16   Report Post  
CJT
 
Posts: n/a
Default

w_tom wrote:
To make it easier for some to understand, I should have first noted
that lightning is an impulse. The classic current transient is an 8/20
microsecond impulse. That impulse is a summation of frequency
components; AC waveforms at various radio frequencies. This is basic
knowledge to those with electrical engineering training AND those with
math backgrounds that include Fourier Series.

Lightning obviously is not DC. If it was, then only wire resistance
would be relevant. An impulse due to lightning contains numeorus AC
sine waves.



That a pulse is susceptible to Fourier analysis does not make it
"contain numerous AC sine waves."

Much of that energy is in the 100 kilohertz and Megahertz
range. These obviously are not DC. Wire resistance has very little
relevance. Wire impedance is a dominant parameter due to sine waves in
an 8/20 usec impulse called CG lightning. Wire impedance would be
completely irrelevant if lightning were DC.

Nothing new about this. First year electrical engineers even study
impulse response. Those who have a problem with this concept will have
difficulty understanding the electrical nature of lightning. Clearly
lightning is not DC because lightning current is an impulse in the
microsecond range.
CJT wrote:
w_tom wrote:
BTW, lightning is not DC. Lightning is AC current - mostly in radio
frequency ranges. This AC nature is why longer conductors, sufficient
for 60 Hz AC electric power, can be poor conductors to lightning; its
energy in higher frequencies. This AC nature of lightning is why so
many humans think lightning is capricious.

Huh? Cite?




--
The e-mail address in our reply-to line is reversed in an attempt to
minimize spam. Our true address is of the form .
  #17   Report Post  
CJT
 
Posts: n/a
Default

w_tom wrote:
Risetime of these current pulses was provided in a previous
post: exponentially rising and falling edges of 8/20
microseconds.


I took what you wrote previously to be the pulse duration, not its
risetime.

Obviously the ground wire would not be run through a coil.
But due to the high frequency nature of transients, that
ground wire must not be spliced, no sharp bends, and not
routed inside metallic materials. A violation would only
increase wire impedance unacceptably. Why? Because even
sharp bends cause significant impedance increases when
discussing the higher frequency components of lightning.


An 8 microsecond risetime doesn't imply all that high a frequency.
Sharp bends in thick wire at ultrasonic frequencies aren't a big deal.

We could play a game of 50,000 questions since even the most
basic nature of lightning - its RF components - is new
information. These and other questions have been answered
previously in a list of citations at:
http://makeashorterlink.com/?X61C23DCA

Figure about one days worth of reading.

CJT wrote:
A small multiple of 40-50 Hz is not exactly RF. So what's the
risetime of these pulses? Duration?
...

Why would you want one to run through a coil, anyway?

I suspect the warning is to avoid somebody being electrocuted by induced
current from the pulse(s).



--
The e-mail address in our reply-to line is reversed in an attempt to
minimize spam. Our true address is of the form .
  #18   Report Post  
w_tom
 
Posts: n/a
Default

You are joking. Right? The Fourier analysis of lightning
reveals energy at numerous frequencies. Lightning is RF
electricity. That RF energy is why lightning even causes
noise on radios. So now you say all those RF sine waves,
demonstrated by Fourier analysis, really do not exist? Why
not just say the world is still flat? If lightning did not
have so much energy in RF, then radios would not receive that
RF noise. An example that demonstrates the RF nature of
pulses and of lightning. Pulses are not DC.

If lightning were DC, then wire impedance would not be
relevant. But lets put numbers to it. A 50 foot 20 amp wire
may be 0.2 ohms resistance. But wire reactance means same
wire has something like 120 ohms impedance to a lightning
transient. Why is impedance so much larger than the
resistance? Because the RF components of lightning make
impedance relevant and significant. If lighting were DC, then
wire resistance and wire impedance would be same. If
lightning were DC, then lightning would not have such
destructive consequences.

Fourier analysis demonstrates lightning is AC electricity at
many frequencies. Putting a number to the impulse - 8/20
microseconds - explains why so much energy is in the Megahertz
range. Using DC analysis to explain lightning means that
person never first learned basic transient analysis taught to
first year EE students. Lightning has massive energy in radio
frequencies which makes lightning so uniquely destructive.
Those radio frequencies even mean that an earthing wire
bundled with other wires will induce transients on those other
wires. DC electricity would not do that inducing.

Why are induced transients a problem with lightning?
Because lighting is AC electricity - as demonstrated by
Fourier analysis, as demonstrated by discussion about
impedance, as demonstrated by what first year EE students are
taught, as demonstrated by noise on the radio, and as
demonstrated by so many industry professional cited in:
http://makeashorterlink.com/?X61C23DCA

Show me. Show me how lightning is only DC. Provide
supporting facts. Provide industry citations as I have done
repeatedly. Declaring something without supporting facts is
classic junk science reasoning. Where are those supporting
facts, the numbers, and industry professional citations that
declare lightning as DC electricity? Without supporting
facts, et al, then your claim is really nothing more than
personal speculation or urban myth. Show me. Provide facts
and numbers. Provide the underlying math. Show me that
capacitors and inductors are irrelevant to conducting
lightning's DC electricity. Show me rather than make
unsubstantiated claims. Show me how you know that lightning
is DC electricity. Show me that you also know the difference
between ultrasonics and electricity.

CJT wrote:
That a pulse is susceptible to Fourier analysis does not make it
"contain numerous AC sine waves."

  #19   Report Post  
w_tom
 
Posts: n/a
Default

Point one. Deny citations from industry professionals by
refusing to read them. An ostrich does that; not an informed
human. Cited are numerous technical papers and other
technical facts on lightning - literally a full days worth of
reading. Instead TimPerry pretends those citations did not
exist in:
http://makeashorterlink.com/?X61C23DCA

A DC pulse is an oxymoron. As confusing as another
ridiculous term: DC triangle wave. Either it is a pulse
affected by reactance (inductors and capacitors), or it is
only DC that completely ignores reactance. One cannot have
it both ways. Either it is DC that ignores reactance, or it
is a pulse that makes reactance relevant. Which is it? If
lightning is DC, then inductance, capacitance, and impedance
is not discussed. Why do industry professionals discuss these
repeatedly? Because lightning is not DC.

Pulses are transient responses - taught in 1st year
engineering - an introduction to AC characteristics. AC
analysis makes wire inductance and capacitance significant.
DC analysis ignores inductance and capacitance. To understand
how lightning works - as cited in
http://makeashorterlink.com/?X61C23DCA - one cannot pretend
lightning is DC. A lightning impulse is an AC or transient
event. Lightning is a composition of many frequencies. Show
me a DC wave that has frequency components? You cannot. That
is the oxymoron of DC pulse. A 'pulse' has AC components.
'DC' has no AC components. Oxymoron.

Equally confusing is to say "DC triangle wave". DC pulse or
DC triangle wave - both are oxymorons. Both contain numerous
frequency components. Therefore a 'DC pulse' cannot be
analyzed using DC analysis. Lightning requires AC analysis.
Lightning is not a DC event.

Point two. TimPerry believes a batting cage set in concrete
is not conductive. TimPerry should first read those
discussions he refused to learn from in:
http://makeashorterlink.com/?X61C23DCA
A most superior earth ground is an Ufer ground. Why? The
earthing is through concrete. Had TimPerry first learned what
industry professionals teach, then he would have known the
batting cage mounted in concrete makes a best earthing ground
- exactly like Ufer grounds. But then TimPerry also promotes
an oxymoron called 'DC pulse'. Somehow he knows without first
learning the facts. If only he had first read those citations
and first learned about Ufer grounding. Then he would not
have posted, "i dont think so." One should first learn before
knowing something.

The energy of lightning is AC electricity.

TimPerry wrote:
citing a newsgroup discussion is not a valid reference.

elaborate (and probably inaccurate) discussion as to the exact
nature of lightning will not be of benefit to the original poster.

a DC pulse is still a DC pulse no amount of wishful thinking will
turn it in to AC (granted that there are some aspects that generate
RF and perhaps some other characteristics that may be able to be
handled by AC analysis)

looking at a few of toms 16,900 archived posts i see he also
claims that a ground rod in rock will make a perfectly good
ground. from that it follows that a batting cage set in concrete
is adiquitly grounded. i dont think so.

  #20   Report Post  
Brian Clark
 
Posts: n/a
Default

w_tom wrote:
You are joking. Right? The Fourier analysis of lightning
reveals energy at numerous frequencies. Lightning is RF
electricity. That RF energy is why lightning even causes
noise on radios. So now you say all those RF sine waves,
demonstrated by Fourier analysis, really do not exist? Why
not just say the world is still flat? If lightning did not
have so much energy in RF, then radios would not receive that
RF noise. An example that demonstrates the RF nature of
pulses and of lightning. Pulses are not DC.

If lightning were DC, then wire impedance would not be
relevant. But lets put numbers to it. A 50 foot 20 amp wire
may be 0.2 ohms resistance. But wire reactance means same
wire has something like 120 ohms impedance to a lightning
transient. Why is impedance so much larger than the
resistance? Because the RF components of lightning make
impedance relevant and significant. If lighting were DC, then
wire resistance and wire impedance would be same. If
lightning were DC, then lightning would not have such
destructive consequences.

Fourier analysis demonstrates lightning is AC electricity at
many frequencies. Putting a number to the impulse - 8/20
microseconds - explains why so much energy is in the Megahertz
range. Using DC analysis to explain lightning means that
person never first learned basic transient analysis taught to
first year EE students. Lightning has massive energy in radio
frequencies which makes lightning so uniquely destructive.
Those radio frequencies even mean that an earthing wire
bundled with other wires will induce transients on those other
wires. DC electricity would not do that inducing.

Why are induced transients a problem with lightning?
Because lighting is AC electricity - as demonstrated by
Fourier analysis, as demonstrated by discussion about
impedance, as demonstrated by what first year EE students are
taught, as demonstrated by noise on the radio, and as
demonstrated by so many industry professional cited in:
http://makeashorterlink.com/?X61C23DCA

Show me. Show me how lightning is only DC. Provide
supporting facts. Provide industry citations as I have done
repeatedly. Declaring something without supporting facts is
classic junk science reasoning. Where are those supporting
facts, the numbers, and industry professional citations that
declare lightning as DC electricity? Without supporting
facts, et al, then your claim is really nothing more than
personal speculation or urban myth. Show me. Provide facts
and numbers. Provide the underlying math. Show me that
capacitors and inductors are irrelevant to conducting
lightning's DC electricity. Show me rather than make
unsubstantiated claims. Show me how you know that lightning
is DC electricity. Show me that you also know the difference
between ultrasonics and electricity.

CJT wrote:

That a pulse is susceptible to Fourier analysis does not make it
"contain numerous AC sine waves."


well I'd never even thought about whether it was AC DC or whatever. But
its an interesting discussion... Its certainly disruptive across a wide
band of the rf spectrum. I wouldn't make a phone call if it was overhead
anyway! hmmm


  #21   Report Post  
CJT
 
Posts: n/a
Default

w_tom wrote:
You are joking. Right? The Fourier analysis of lightning
reveals energy at numerous frequencies. Lightning is RF
electricity. That RF energy is why lightning even causes
noise on radios. So now you say all those RF sine waves,
demonstrated by Fourier analysis, really do not exist?


You're demonstrating a basic lack of understanding of
Fourier analysis. Fourier used the sine function as a
basis function. That doesn't mean there are sine waves
"in" the pulse. If he had used some other set of basis
functions, would that "prove" that those other bases were
present in the pulse?

Don't get me wrong -- I'm ok with you saying there's RF
energy in the pulse, but not with your extrapolation to
"RF sine waves." We're talking about impulses here.

Why
not just say the world is still flat? If lightning did not
have so much energy in RF, then radios would not receive that
RF noise.


The radios are responding to the pulses. Fourier series are a
useful way to analyze the response. But the radio is essentially
a filter that alters the signal it sees. It's not locking on to
some RF sine wave as you seem to think.

An example that demonstrates the RF nature of
pulses and of lightning. Pulses are not DC.

If lightning were DC, then wire impedance would not be
relevant. But lets put numbers to it. A 50 foot 20 amp wire
may be 0.2 ohms resistance. But wire reactance means same
wire has something like 120 ohms impedance to a lightning
transient.


Sure -- put numbers to it. But show your work.

Why is impedance so much larger than the
resistance? Because the RF components of lightning make
impedance relevant and significant. If lighting were DC, then
wire resistance and wire impedance would be same. If
lightning were DC, then lightning would not have such
destructive consequences.

Huh? Have you seen those videos of (as I recall) DC arcs in
high voltage transmission tests that are floating around the Web?

Fourier analysis demonstrates lightning is AC electricity at
many frequencies. Putting a number to the impulse - 8/20
microseconds - explains why so much energy is in the Megahertz
range.


That's why I asked you for risetime numbers. Show your work.
What fraction of the energy is in the MHz range?

Using DC analysis to explain lightning means that
person never first learned basic transient analysis taught to
first year EE students.


I haven't noticed anyone here advocating the use of DC analysis
per se, so that's a red herring.

Lightning has massive energy in radio
frequencies which makes lightning so uniquely destructive.
Those radio frequencies even mean that an earthing wire
bundled with other wires will induce transients on those other
wires. DC electricity would not do that inducing.


Think pulse.

Why are induced transients a problem with lightning?
Because lighting is AC electricity - as demonstrated by
Fourier analysis, as demonstrated by discussion about
impedance, as demonstrated by what first year EE students are
taught, as demonstrated by noise on the radio, and as
demonstrated by so many industry professional cited in:
http://makeashorterlink.com/?X61C23DCA

Show me. Show me how lightning is only DC.


Who's that addressed to? I never said lightning is only DC.
I did say you haven't established it's "AC [or RF] sine waves."

Provide
supporting facts. Provide industry citations as I have done
repeatedly. Declaring something without supporting facts is
classic junk science reasoning. Where are those supporting
facts, the numbers, and industry professional citations that
declare lightning as DC electricity? Without supporting
facts, et al, then your claim is really nothing more than
personal speculation or urban myth. Show me. Provide facts
and numbers. Provide the underlying math. Show me that
capacitors and inductors are irrelevant to conducting
lightning's DC electricity. Show me rather than make
unsubstantiated claims. Show me how you know that lightning
is DC electricity. Show me that you also know the difference
between ultrasonics and electricity.

CJT wrote:
That a pulse is susceptible to Fourier analysis does not make it
"contain numerous AC sine waves."



--
The e-mail address in our reply-to line is reversed in an attempt to
minimize spam. Our true address is of the form .
  #22   Report Post  
w_tom
 
Posts: n/a
Default

The radio is tuned only to receive a specific frequency.
That pulse is composed of many frequencies. Frequency that
interferes with radio reception is a unique sine wave
frequency
within that pulse. Just one of so many frequencies that
create the pulse. Pulses - like all waveforms - are a
summation of basic sine waves at different frequencies,
amplitudes, and phases. The radio does not receive a pulse.
It receives only parts of a waveform that are specific to its
tuned frequency. Fourier analysis demonstrates the concept.
The radio does not receive a pulse so much as it receives one
frequency that was part of that pulse.

A chart for the frequency spectrum of lightning is available
in:
http://www.hubersuhner.com/products/...kb-bas-fre.htm
..

What does DC arcing create? That was how radios worked.
Sparking DC electricity created AC electricity that resulted
in radio waves. Telsa did not transmit DC electricity. To
perform electricity transmissions, DC electricity was
converted to AC. DC arcing is detected how? By measuring AC
components created by that arcing.

Meanwhile TimPerry repeatedly refers to DC pulses. That
would be DC analysis. Lightning pulse is AC electricity. It
creates electromagnetic waves of same frequencies. You even
have a chart for that frequency spectrum. What the chart does
not show is how the energy content quickly tapers to zero as
frequency approaches DC. That pulse called lightning is
composed of electricity at RF frequencies.

CJT wrote:
w_tom wrote:
... Lightning is RF electricity. That RF energy is why
lightning even causes noise on radios. So now you say
all those RF sine waves, demonstrated by Fourier
analysis, really do not exist?


You're demonstrating a basic lack of understanding of
Fourier analysis. Fourier used the sine function as a
basis function. That doesn't mean there are sine waves
"in" the pulse. If he had used some other set of basis
functions, would that "prove" that those other bases were
present in the pulse?

Don't get me wrong -- I'm ok with you saying there's RF
energy in the pulse, but not with your extrapolation to
"RF sine waves." We're talking about impulses here.
...

The radios are responding to the pulses. Fourier series are a
useful way to analyze the response. But the radio is essentially
a filter that alters the signal it sees. It's not locking on to
some RF sine wave as you seem to think.
...

Huh? Have you seen those videos of (as I recall) DC arcs in
high voltage transmission tests that are floating around the Web?

  #23   Report Post  
CJT
 
Posts: n/a
Default

w_tom wrote:
The radio is tuned only to receive a specific frequency.
That pulse is composed of many frequencies.


The pulse is a pulse.

Frequency that
interferes with radio reception is a unique sine wave
frequency
within that pulse.


You're really hung up on those sine waves for some reason.
Fourier analysis is a mathematical construct (admittedly
a very useful one), but it's a fiction (in the best sense
of the word). The lightning isn't "created" from sine
waves.

Just one of so many frequencies that
create the pulse. Pulses - like all waveforms - are a
summation of basic sine waves at different frequencies,
amplitudes, and phases.


So what's the phase of the third sine wave in the series?
What physical phenomenon results in that phase? Does the
Fourier series you contend results in the single lightning
pulse match it through all time? Think about what you're
saying.

The radio does not receive a pulse.


Huh? Why not?

It receives only parts of a waveform that are specific to its
tuned frequency. Fourier analysis demonstrates the concept.
The radio does not receive a pulse so much as it receives one
frequency that was part of that pulse.


So what magic keeps the other parts of the pulse away from the radio?
Face it, the radio gets the full pulse and processes it into whatever
you hear.


A chart for the frequency spectrum of lightning is available
in:
http://www.hubersuhner.com/products/...kb-bas-fre.htm
.


Did you look at the scale on that chart? Is that your idea of RF?
Looks more like audio to me.

What does DC arcing create? That was how radios worked.
Sparking DC electricity created AC electricity that resulted
in radio waves. Telsa did not transmit DC electricity. To
perform electricity transmissions, DC electricity was
converted to AC. DC arcing is detected how? By measuring AC
components created by that arcing.



You're arguing with yourself. I didn't say it was DC (or AC for
that matter -- the DC/AC distinction, as explored recently in
another thread, is not necessarily useful in a discussion such
as this).


Meanwhile TimPerry repeatedly refers to DC pulses. That
would be DC analysis. Lightning pulse is AC electricity. It
creates electromagnetic waves of same frequencies. You even
have a chart for that frequency spectrum. What the chart does
not show is how the energy content quickly tapers to zero as
frequency approaches DC. That pulse called lightning is
composed of electricity at RF frequencies.


I suspect we're more in agreement than would appear. But my problem
with your position is twofold:

- your insistence that a pulse in nature is somehow created from lots
of sine waves; maybe it's just the words you use, but I have a
philosophical problem with projecting a mathematical analysis
technique onto nature as more than an analogy

- (not explored here yet, but related) your claim in one of your
posts that a sharp bend in a ground wire would necessarily destroy
its effectiveness in shunting lightning to ground because of effects
on its impedance. I think you also contend that wrapping a coil
around the wire would cause a similar problem. I assume you would
then contend that all skyscrapers run their lightning rod ground wires
outside their girder structures, to avoid being encompassed by single
turn coils consisting of metallic girders.

CJT wrote:
w_tom wrote:
... Lightning is RF electricity. That RF energy is why
lightning even causes noise on radios. So now you say
all those RF sine waves, demonstrated by Fourier
analysis, really do not exist?

You're demonstrating a basic lack of understanding of
Fourier analysis. Fourier used the sine function as a
basis function. That doesn't mean there are sine waves
"in" the pulse. If he had used some other set of basis
functions, would that "prove" that those other bases were
present in the pulse?

Don't get me wrong -- I'm ok with you saying there's RF
energy in the pulse, but not with your extrapolation to
"RF sine waves." We're talking about impulses here.
...

The radios are responding to the pulses. Fourier series are a
useful way to analyze the response. But the radio is essentially
a filter that alters the signal it sees. It's not locking on to
some RF sine wave as you seem to think.
...

Huh? Have you seen those videos of (as I recall) DC arcs in
high voltage transmission tests that are floating around the Web?



--
The e-mail address in our reply-to line is reversed in an attempt to
minimize spam. Our true address is of the form .
  #24   Report Post  
CJT
 
Posts: n/a
Default

w_tom wrote:
The radio is tuned only to receive a specific frequency.
That pulse is composed of many frequencies. Frequency that
interferes with radio reception is a unique sine wave
frequency
within that pulse. Just one of so many frequencies that
create the pulse. Pulses - like all waveforms - are a
summation of basic sine waves at different frequencies,
amplitudes, and phases. The radio does not receive a pulse.
It receives only parts of a waveform that are specific to its
tuned frequency. Fourier analysis demonstrates the concept.
The radio does not receive a pulse so much as it receives one
frequency that was part of that pulse.

A chart for the frequency spectrum of lightning is available
in:
http://www.hubersuhner.com/products/...kb-bas-fre.htm

snip

BTW, here's another page in that same series:
http://www.hubersuhner.com/mozilla/p...-bas-espec.htm

I guess you would have to claim there's positive AC and negative AC to
be consistent with it.

--
The e-mail address in our reply-to line is reversed in an attempt to
minimize spam. Our true address is of the form .
  #25   Report Post  
w_tom
 
Posts: n/a
Default

You are hung up on a pulse. True, the sine waves that
combine to create a pulse exist with boundary conditions. A
true sine wave goes forever - has no boundary conditions. But
how do we measure the frequency response of circuits? We
apply a signal that is chock full of 'sine waves' at various
frequencies (for a limited time - the boundary condition) and
then learn which sine waves come out the other end. We apply a
pulse. We measure those 'sine waves' with a spectrum
analyzer. If pulses did not create sine waves at all those
frequencies, then the spectrum analyzer would do nothing
useful. Just another example of how a pulse is far more than
just a pulse.

A radio receives electromagnetic radiation at its tuned
frequency (assuming the signal is not so strong as to
overwhelm filters). If a pulse is only a pulse then the radio
does not receive anything. But since a pulse is AC
electricity at numerous frequencies, then the radio receives
only its unique frequency from that pulse.

I made it easy. Chart provides a frequency spectrum for a
lightning pulse. How can the pulse have a frequency spectrum
if a pulse is only a pulse? Yes some of those RF frequencies
are so low as to be same as audio frequencies. But they are
electrical - not mechanical motion. The electricity is still
RF frequencies. Some of the most destructive energy is found
in those higher (and less amplitude) frequencies.

I guess you are finally coming to accept the fact that a
lightning pulse is not just a pulse (and not a DC pulse as
TimPerry claimed). You don't provide any supporting facts,
numbers, or citations for your claim that "a pulse is only a
pulse". Provided were numerous reasons why a lightning pulse
is composed of AC electricity at numerous RF frequencies.
Numerous sine waves summed together to create a pulse. To
repeat some supporting facts: We measure the frequency
spectrum of a pulse because a pulse is composed of so many
different frequencies. That lightning pulse is affected by
impedance because it contains radio frequencies - not DC. The
pulse contains frequencies that create noise on tuned radios.
Wire impedance created by sharp wire bends can undermine a
protection system because lightning is composed of AC
electricity. A chart with the frequency spectrum for a
lightning pulse is provided showing energy even at the
megahertz range. The so called DC arc is really AC
electricity. Even Fourier analysis demonstrates that all
waveforms (such as pulses) are summations of sine waves at
various amplitudes, frequencies, and phases. Lighting pulse
contains many sine waves (with boundary conditions). The
sharper that pulse, then the more frequencies are contained in
a pulse.

Meanwhile your concept of earthing a steel building is
flawed. The steel frame is sufficient to be an earth ground.
It does not have high impedance to obstruct a lightning
strike. However, for better protection, the lightning rod is
earthed using wire outside the building. Better protection
means a building's earthing meets lightning rod's earthing at
a point beneath the building - the single point grounding
concept. A building structure is not some big 'lightning
impeding' inductor. However even wire has impedance which is
why shorter connections to earth ground mean superior
lightning protection. Wire impedance is also why plug-in
protectors are not effectively earthed.

Lastly cited is how BT and other telcos earth their
switching stations to not suffer lightning damage. Every
incoming wire is earthed ideally 50 meters before those wires
connect to the computer. Connection from each incoming wire
to earth is as short as possible. A larger separation between
'earthing connection to computer' provides more impedance -
better computer protection. Short connection 'from incoming
wire to earth' means less impedance - a better path for
lightning. Why do we mention this? Because lightning
protection is about low impedance earthing. Better lightning
protection means the protector is where utility wires enter
the building, with a shortest connection to a single point
earth ground, and not adjacent to electronics.

A lightning pulse is not just a pulse. Lightning is many AC
waves in a wide frequency spectrum - AC electricity - that
seeks earth ground. Protection from lightning involves RF
principles that radio engineers best appreciate.

CJT wrote:
w_tom wrote:
The radio is tuned only to receive a specific frequency.
That pulse is composed of many frequencies.


The pulse is a pulse.

Frequency that
interferes with radio reception is a unique sine wave
frequency
within that pulse.


You're really hung up on those sine waves for some reason.
Fourier analysis is a mathematical construct (admittedly
a very useful one), but it's a fiction (in the best sense
of the word). The lightning isn't "created" from sine
waves.

Just one of so many frequencies that
create the pulse. Pulses - like all waveforms - are a
summation of basic sine waves at different frequencies,
amplitudes, and phases.


So what's the phase of the third sine wave in the series?
What physical phenomenon results in that phase? Does the
Fourier series you contend results in the single lightning
pulse match it through all time? Think about what you're
saying.

The radio does not receive a pulse.


Huh? Why not?

It receives only parts of a waveform that are specific to its
tuned frequency. Fourier analysis demonstrates the concept.
The radio does not receive a pulse so much as it receives one
frequency that was part of that pulse.


So what magic keeps the other parts of the pulse away from the radio?
Face it, the radio gets the full pulse and processes it into whatever
you hear.


A chart for the frequency spectrum of lightning is available
in:
http://www.hubersuhner.com/products/...kb-bas-fre.htm
.


Did you look at the scale on that chart? Is that your idea of RF?
Looks more like audio to me.

What does DC arcing create? That was how radios worked.
Sparking DC electricity created AC electricity that resulted
in radio waves. Telsa did not transmit DC electricity. To
perform electricity transmissions, DC electricity was
converted to AC. DC arcing is detected how? By measuring AC
components created by that arcing.


You're arguing with yourself. I didn't say it was DC (or AC for
that matter -- the DC/AC distinction, as explored recently in
another thread, is not necessarily useful in a discussion such
as this).


Meanwhile TimPerry repeatedly refers to DC pulses. That
would be DC analysis. Lightning pulse is AC electricity. It
creates electromagnetic waves of same frequencies. You even
have a chart for that frequency spectrum. What the chart does
not show is how the energy content quickly tapers to zero as
frequency approaches DC. That pulse called lightning is
composed of electricity at RF frequencies.


I suspect we're more in agreement than would appear. But my problem
with your position is twofold:

- your insistence that a pulse in nature is somehow created from lots
of sine waves; maybe it's just the words you use, but I have a
philosophical problem with projecting a mathematical analysis
technique onto nature as more than an analogy

- (not explored here yet, but related) your claim in one of your
posts that a sharp bend in a ground wire would necessarily destroy
its effectiveness in shunting lightning to ground because of effects
on its impedance. I think you also contend that wrapping a coil
around the wire would cause a similar problem. I assume you would
then contend that all skyscrapers run their lightning rod ground wires
outside their girder structures, to avoid being encompassed by single
turn coils consisting of metallic girders.



  #26   Report Post  
TimPerry
 
Posts: n/a
Default


"w_tom" wrote in message
...
Point one. Deny citations from industry professionals by
refusing to read them. An ostrich does that; not an informed
human. Cited are numerous technical papers and other
technical facts on lightning - literally a full days worth of
reading. Instead TimPerry pretends those citations did not
exist in:
http://makeashorterlink.com/?X61C23DCA


one has no way of knowing who is an "industry professional" and who is a
yoyo.


A DC pulse is an oxymoron. As confusing as another
ridiculous term: DC triangle wave. Either it is a pulse
affected by reactance (inductors and capacitors), or it is
only DC that completely ignores reactance. One cannot have
it both ways. Either it is DC that ignores reactance, or it
is a pulse that makes reactance relevant. Which is it? If
lightning is DC, then inductance, capacitance, and impedance
is not discussed. Why do industry professionals discuss these
repeatedly? Because lightning is not DC.

Pulses are transient responses - taught in 1st year
engineering - an introduction to AC characteristics. AC
analysis makes wire inductance and capacitance significant.
DC analysis ignores inductance and capacitance. To understand
how lightning works - as cited in
http://makeashorterlink.com/?X61C23DCA - one cannot pretend
lightning is DC. A lightning impulse is an AC or transient
event. Lightning is a composition of many frequencies. Show
me a DC wave that has frequency components? You cannot. That
is the oxymoron of DC pulse. A 'pulse' has AC components.
'DC' has no AC components. Oxymoron.

Equally confusing is to say "DC triangle wave"


nothing confusing about it to me. if a waveform lies above reference ground
it becomes fluctuating DC regardless of its shape, frequency, pulse
repetition rate. id just call it a triangle wave or sawtooth or whatever.
the DC part is usully understood.

you see the electrons are all moving in one direction through a conductor...
that's what Direct Current is. when the little suckers stop and reverse
direction on a regular basis then voila! we get Alternating Current.




.. DC pulse or
DC triangle wave - both are oxymorons. Both contain numerous
frequency components. Therefore a 'DC pulse' cannot be
analyzed using DC analysis. Lightning requires AC analysis.
Lightning is not a DC event.


shure is. next thing you know you'll be telling me that a flashlight is AC
because when you turn it on it has a fast rise time, a finite duration, and
a falling edge when you turn it off.... one big square wave.... plus a few
harmonics... and a tiny bit of RF.



Point two. TimPerry believes a batting cage set in concrete
is not conductive.


yep that's what i believe. assuming a typical chain link fence type
arangement. granted that any insulator has a breakdown point. i beleive that
a barefoot boy leaning aginst a batting cage in a thunderstorm is a bad
thing. even a boy in wet sneakers.

now should said cage be grounded with standard 8 foot rods at each corner
and possibly at intermediate points i might feel differently. its just that
i have never seen one treated in such a way. (remember we are talking little
league here). even in this case i wouldnt be comfortable. a direct hit by
approx 20,000 amps is not something i would care to experiance given the
choice.
in any event, i personally prefer to be at home or on a nice building during
lightning storms. after the storm has passed i then go to various locations
as needed and reset breakers, make repairs, or go to backup systems.



TimPerry should first read those
discussions he refused to learn from in:
http://makeashorterlink.com/?X61C23DCA


sorry, if i cite a reference it will be to an accredited text, validated
research paper, or at least a website from a reasonably reputable
organization or agency.



A most superior earth ground is an Ufer ground. Why? The
earthing is through concrete. Had TimPerry first learned what
industry professionals teach, then he would have known the
batting cage mounted in concrete makes a best earthing ground
- exactly like Ufer grounds. But then TimPerry also promotes
an oxymoron called 'DC pulse'. Somehow he knows without first
learning the facts. If only he had first read those citations
and first learned about Ufer grounding. Then he would not
have posted, "i dont think so." One should first learn before
knowing something.

The energy of lightning is AC electricity.

TimPerry wrote:
citing a newsgroup discussion is not a valid reference.

elaborate (and probably inaccurate) discussion as to the exact
nature of lightning will not be of benefit to the original poster.

a DC pulse is still a DC pulse no amount of wishful thinking will
turn it in to AC (granted that there are some aspects that generate
RF and perhaps some other characteristics that may be able to be
handled by AC analysis)

looking at a few of toms 16,900 archived posts i see he also
claims that a ground rod in rock will make a perfectly good
ground. from that it follows that a batting cage set in concrete
is adiquitly grounded. i dont think so.





  #27   Report Post  
Leonard Caillouet
 
Posts: n/a
Default

w_tom,

While you are correct in many ways, CJT is also correct in pointing out that
your suggestion that lightning is made of sine waves is faulty application
of Fourier. You could just as easily say that lightning is made of wavelets
(and more correctly, actually) or many other functions. These are all just
mathematical constructs to describe a complex phenomenon as components that
can be manipulated for analysis. The point that lightning is a pulse and
can be analyzed by its component frequencies should be clear. That it is
"made of sine waves" is an equally clearly faulty application of the
concept. If you would learn to be a bit more humble in accepting criticism
of your language, useage, and out of context application of concepts, the
correct basis of your arguments might be more often appreciated. Mostly,
you end up looking like an idiot.

Leonard

"w_tom" wrote in message
...
You are hung up on a pulse. True, the sine waves that
combine to create a pulse exist with boundary conditions. A
true sine wave goes forever - has no boundary conditions. But
how do we measure the frequency response of circuits? We
apply a signal that is chock full of 'sine waves' at various
frequencies (for a limited time - the boundary condition) and
then learn which sine waves come out the other end. We apply a
pulse. We measure those 'sine waves' with a spectrum
analyzer. If pulses did not create sine waves at all those
frequencies, then the spectrum analyzer would do nothing
useful. Just another example of how a pulse is far more than
just a pulse.



  #28   Report Post  
Asimov
 
Posts: n/a
Default

"w_tom" bravely wrote to "All" (07 Jul 05 22:29:12)
--- on the heady topic of " Lightning protection"

w_ From: w_tom
w_ Xref: aeinews sci.electronics.repair:52721

w_ Meanwhile TimPerry repeatedly refers to DC pulses. That
w_ would be DC analysis. Lightning pulse is AC electricity. It
w_ creates electromagnetic waves of same frequencies. You even
w_ have a chart for that frequency spectrum. What the chart does
w_ not show is how the energy content quickly tapers to zero as
w_ frequency approaches DC. That pulse called lightning is
w_ composed of electricity at RF frequencies.

He is not completely wrong, Tom. There is a phenomenon of charge
separation which takes place in the cloud and there is a DC potential.
The initial discharge starts with DC but with distance and motion we
have the effects of inductance and capacitance which establish an
impedance and results in reflections where discontinuities exist for a
moving wavefront. What we basically have is a square wave going from
one DC potential to ground but, like the Fourrier model, made up of an
infinite series of sine waves. So yes, there will be AC but the
initial natural process is all about charge separation in the clouds.
It can be no other way.

A*s*i*m*o*v

.... Power is obtained by current meeting resistance

  #29   Report Post  
CJT
 
Posts: n/a
Default

w_tom wrote:
You are hung up on a pulse. True, the sine waves that
combine to create a pulse exist with boundary conditions. A
true sine wave goes forever - has no boundary conditions. But
how do we measure the frequency response of circuits? We
apply a signal that is chock full of 'sine waves' at various
frequencies (for a limited time - the boundary condition) and
then learn which sine waves come out the other end. We apply a
pulse. We measure those 'sine waves' with a spectrum
analyzer. If pulses did not create sine waves at all those
frequencies, then the spectrum analyzer would do nothing
useful. Just another example of how a pulse is far more than
just a pulse.



Certainly pulses can excite tuned circuits. That's different
from what you have been saying.

A radio receives electromagnetic radiation at its tuned
frequency (assuming the signal is not so strong as to
overwhelm filters). If a pulse is only a pulse then the radio
does not receive anything. But since a pulse is AC
electricity at numerous frequencies, then the radio receives
only its unique frequency from that pulse.

I made it easy. Chart provides a frequency spectrum for a
lightning pulse. How can the pulse have a frequency spectrum
if a pulse is only a pulse? Yes some of those RF frequencies
are so low as to be same as audio frequencies. But they are
electrical - not mechanical motion. The electricity is still
RF frequencies. Some of the most destructive energy is found
in those higher (and less amplitude) frequencies.


You seem to be using "RF frequencies" to mean "electromagnetic
radiation." I think that's another example of imprecise language.

I guess you are finally coming to accept the fact that a
lightning pulse is not just a pulse (and not a DC pulse as
TimPerry claimed). You don't provide any supporting facts,
numbers, or citations for your claim that "a pulse is only a
pulse". Provided were numerous reasons why a lightning pulse
is composed of AC electricity at numerous RF frequencies.


I've deliberately steered clear of the DC vs AC controversy.
If pressed, I'd probably come down on the side of TimPerry,
because I find his flashlight analogy persuasive. But I don't
think the distinction is particularly useful, so I find it
uninteresting. You and TimPerry could probably resolve your
differences by careful definition of the terms DC and AC. I
suspect that under your definition, there has never been a true
DC (i.e. unvarying, as I understand your position) source.

Numerous sine waves summed together to create a pulse. To
repeat some supporting facts: We measure the frequency
spectrum of a pulse because a pulse is composed of so many
different frequencies. That lightning pulse is affected by
impedance because it contains radio frequencies - not DC. The
pulse contains frequencies that create noise on tuned radios.
Wire impedance created by sharp wire bends can undermine a
protection system because lightning is composed of AC
electricity.


Without some calculation to support it, that's a vacuous
statement. I think it's probably also incorrect in general.

You cited a surge arrestor company's Web site earlier. Do
you think the ground wires associated with surge arrestors
must similarly avoid sharp bends?

A chart with the frequency spectrum for a
lightning pulse is provided showing energy even at the
megahertz range. The so called DC arc is really AC
electricity. Even Fourier analysis demonstrates that all
waveforms (such as pulses) are summations of sine waves at
various amplitudes, frequencies, and phases.


At the risk of sounding too much like a former President, that
depends on your definition of "are." "Can be analyzed for
certain purposes as if they were" might be closer to the truth.

Lighting pulse
contains many sine waves (with boundary conditions). The
sharper that pulse, then the more frequencies are contained in
a pulse.

Meanwhile your concept of earthing a steel building is
flawed. The steel frame is sufficient to be an earth ground.
It does not have high impedance to obstruct a lightning
strike. However, for better protection, the lightning rod is
earthed using wire outside the building. Better protection
means a building's earthing meets lightning rod's earthing at
a point beneath the building - the single point grounding
concept. A building structure is not some big 'lightning
impeding' inductor. However even wire has impedance which is
why shorter connections to earth ground mean superior
lightning protection. Wire impedance is also why plug-in
protectors are not effectively earthed.


Apparently I was unclear. :-) I'm not aware of any requirement
that the wire grounding a lightning rod not be encircled at any
point by the building's girders. Another post of yours seemed
to imply such a requirement.

Lastly cited is how BT and other telcos earth their
switching stations to not suffer lightning damage. Every
incoming wire is earthed ideally 50 meters before those wires
connect to the computer. Connection from each incoming wire
to earth is as short as possible. A larger separation between
'earthing connection to computer' provides more impedance -
better computer protection. Short connection 'from incoming
wire to earth' means less impedance - a better path for
lightning. Why do we mention this? Because lightning
protection is about low impedance earthing. Better lightning
protection means the protector is where utility wires enter
the building, with a shortest connection to a single point
earth ground, and not adjacent to electronics.

A lightning pulse is not just a pulse. Lightning is many AC
waves in a wide frequency spectrum - AC electricity - that
seeks earth ground. Protection from lightning involves RF
principles that radio engineers best appreciate.

CJT wrote:
w_tom wrote:
The radio is tuned only to receive a specific frequency.
That pulse is composed of many frequencies.

The pulse is a pulse.

Frequency that
interferes with radio reception is a unique sine wave
frequency
within that pulse.

You're really hung up on those sine waves for some reason.
Fourier analysis is a mathematical construct (admittedly
a very useful one), but it's a fiction (in the best sense
of the word). The lightning isn't "created" from sine
waves.

Just one of so many frequencies that
create the pulse. Pulses - like all waveforms - are a
summation of basic sine waves at different frequencies,
amplitudes, and phases.

So what's the phase of the third sine wave in the series?
What physical phenomenon results in that phase? Does the
Fourier series you contend results in the single lightning
pulse match it through all time? Think about what you're
saying.

The radio does not receive a pulse.

Huh? Why not?

It receives only parts of a waveform that are specific to its
tuned frequency. Fourier analysis demonstrates the concept.
The radio does not receive a pulse so much as it receives one
frequency that was part of that pulse.

So what magic keeps the other parts of the pulse away from the radio?
Face it, the radio gets the full pulse and processes it into whatever
you hear.

A chart for the frequency spectrum of lightning is available
in:
http://www.hubersuhner.com/products/...kb-bas-fre.htm
.

Did you look at the scale on that chart? Is that your idea of RF?
Looks more like audio to me.

What does DC arcing create? That was how radios worked.
Sparking DC electricity created AC electricity that resulted
in radio waves. Telsa did not transmit DC electricity. To
perform electricity transmissions, DC electricity was
converted to AC. DC arcing is detected how? By measuring AC
components created by that arcing.

You're arguing with yourself. I didn't say it was DC (or AC for
that matter -- the DC/AC distinction, as explored recently in
another thread, is not necessarily useful in a discussion such
as this).

Meanwhile TimPerry repeatedly refers to DC pulses. That
would be DC analysis. Lightning pulse is AC electricity. It
creates electromagnetic waves of same frequencies. You even
have a chart for that frequency spectrum. What the chart does
not show is how the energy content quickly tapers to zero as
frequency approaches DC. That pulse called lightning is
composed of electricity at RF frequencies.

I suspect we're more in agreement than would appear. But my problem
with your position is twofold:

- your insistence that a pulse in nature is somehow created from lots
of sine waves; maybe it's just the words you use, but I have a
philosophical problem with projecting a mathematical analysis
technique onto nature as more than an analogy

- (not explored here yet, but related) your claim in one of your
posts that a sharp bend in a ground wire would necessarily destroy
its effectiveness in shunting lightning to ground because of effects
on its impedance. I think you also contend that wrapping a coil
around the wire would cause a similar problem. I assume you would
then contend that all skyscrapers run their lightning rod ground wires
outside their girder structures, to avoid being encompassed by single
turn coils consisting of metallic girders.



--
The e-mail address in our reply-to line is reversed in an attempt to
minimize spam. Our true address is of the form .
  #30   Report Post  
w_tom
 
Posts: n/a
Default

The term DC pulses is an oxymoron. However Asimov is
correct. There must be a DC component to a frequency spectrum
for current. Problem is that most professional sources say
the frequency spectrum for that lightning pulse drops off as
frequency decreases. Some put lower frequency limits at 1
Hz. Others at 0.1 Hz. IOW the DC component that does exist
would be too small to be considered.

Again I don't really understand why this would be true.
When the pulse is done, positive and negative charges have
been neutralized by a unidirectional flow of electrons. The
frequency spectrum from http://www.hubersuhner.com does not
show this fall off at lowest frequencies. But the chart also
does not show frequencies below 1 Hz.

At any rate, perspective of this discussion is Lightning
protection. (Although it originally was about a faraday cage
provided by a chain link cage), it has morphed into the AC
nature of lightning; the component that makes lightning so
challenging and destructive. So challenging that some call
lightning capricious. The AC component in lightning that make
protection difficult is not intuitively obvious. The
components that tend to be most challenging are in the
kilohertz and megahertz range. IOW the DC component is rarely
considered in the overall design of lightning protection.
Higher frequency components are discussed as if higher
frequencies are the only part that exists.

Asimov wrote:
He is not completely wrong, Tom. There is a phenomenon of charge
separation which takes place in the cloud and there is a DC potential.
The initial discharge starts with DC but with distance and motion we
have the effects of inductance and capacitance which establish an
impedance and results in reflections where discontinuities exist for a
moving wavefront. What we basically have is a square wave going from
one DC potential to ground but, like the Fourrier model, made up of an
infinite series of sine waves. So yes, there will be AC but the
initial natural process is all about charge separation in the clouds.
It can be no other way.



  #31   Report Post  
w_tom
 
Posts: n/a
Default

Yo-yos are easily separated from industry professionals.
Yo-yos post claims without numbers, supporting facts, and
experimental evidence (examples). IOW they post junk
science. Junk scientists especially fear to provide numbers.
Why? Numbers identify myth purveyors from the educated. To
promote myths, one must not provide numbers that can be
challenged.

For example, others may recommend an APC power strip
protector. They claim it will block or stop surges? So how
does a transient that was not stopped by 3 miles of air get
stopped by a one inch part? More damning numbers. The
manufacturer's own specifications also make that obvious.
Therefore yo-yos avoid saying what the plug-in protector even
does. They don't cite joules (a number). They don't cite the
manufacturer's specifications. They don't have any idea how
its internal components (MOVs) operate. Some are so foolishly
deceived into thinking MOVs vaporize (fail catastrophically)
to provide protection. They don't even know the so called
protected appliance still connects directly to AC mains -
nothing between the appliance and wall receptacle wire. They
claim a ground light is reporting the existence of an earth
ground. They claim the OK light reports the protector as
fully functional.

So that yo-yos are not exposed, they don't provide any
supporting facts and numbers. Notice why my every post is so
long? Numerous supporting facts, electrical principles,
numbers, and examples are also provided.

Its a classic lesson from propaganda experts. Declare the
other as wrong AND say no more. The naive among us will then
believe the shortest post that only says, "wrong" rather than
the longer post that provides supporting reasons why. Some
people will always believe a sound byte and deny even
principles taught in high school science.

Numerous responsible industry professionals were cited in
that discussion. Everything from Sun Microsystem's Server
Installation manual, National Electrical Code, and even peer
reviewed IEEE papers. The benchmark in protection is
Polyphaser. Their application notes are legendary. And if
that were not enough, visit a newsgroup where lightning is
a most serious problem - where direct lightning strikes are
routinely earthed without damage:
rec.radio.amateur.antenna

Those with technical knowledge will appreciate the
underlying concepts, solutions, and personal experience
provided by posters such as Richard Harrison and Jack
Painter. Principles that were even demonstrated by Ben
Franklin in 1752. Yo-yos, however, will only make
declarations; never provide critical supporting facts,
principles, numbers, and real world experience.

Ironic that TimPerry would say DC triangle waves are not
confusing since that is not what he said on 11 Jun 2005 in the
newsgroup sci.electronics.basics entitled "DC Wave
Questions":
There are a number of waveforms that go in only one
direction relative to ground such as sawtooth waves, square
waves, and triangle waves. To refer to these as say a "DC
triangle wave" would be equally confusing without further
qualification.


Meanwhile, concrete is not an insulator as TimPerry would
have us believe. Concrete is a conductor. Concrete is why
Ufer grounds are a most effective method of protecting
transistors ... or protecting munitions (the original purpose
of Ufer grounds).

TimPerry wrote:
"w_tom" wrote in message
...
http://makeashorterlink.com/?X61C23DCA


one has no way of knowing who is an "industry professional" and
who is a yoyo.
...

Equally confusing is to say "DC triangle wave"


nothing confusing about it to me. if a waveform lies above
reference ground it becomes fluctuating DC regardless of
its shape, frequency, pulse repetition rate. id just call
it a triangle wave or sawtooth or whatever. the DC part is
usully understood.

you see the electrons are all moving in one direction through
a conductor... that's what Direct Current is. when the
little suckers stop and reverse direction on a regular basis
then voila! we get Alternating Current.

DC pulse or DC triangle wave - both are oxymorons. Both
contain numerous frequency components. Therefore a
'DC pulse' cannot be analyzed using DC analysis. Lightning
requires AC analysis. Lightning is not a DC event.


shure is. next thing you know you'll be telling me that a
flashlight is AC because when you turn it on it has a fast
rise time, a finite duration, and a falling edge when you
turn it off.... one big square wave.... plus a few
harmonics... and a tiny bit of RF.

Point two. TimPerry believes a batting cage set in
concrete is not conductive.


yep that's what i believe. assuming a typical chain link fence
type arangement. granted that any insulator has a breakdown
point. i beleive that a barefoot boy leaning aginst a batting
cage in a thunderstorm is a bad thing. even a boy in wet
sneakers.

now should said cage be grounded with standard 8 foot rods at
each corner and possibly at intermediate points i might feel
differently. its just that i have never seen one treated in
such a way. (remember we are talking little league here). even
in this case i wouldnt be comfortable. a direct hit by approx
20,000 amps is not something i would care to experiance given
the choice. in any event, i personally prefer to be at home
or on a nice building during lightning storms. after the
storm has passed i then go to various locations as needed and
reset breakers, make repairs, or go to backup systems.

TimPerry should first read those discussions he refused
to learn from in:
http://makeashorterlink.com/?X61C23DCA


sorry, if i cite a reference it will be to an accredited
text, validated research paper, or at least a website from
a reasonably reputable organization or agency.
...

  #32   Report Post  
w_tom
 
Posts: n/a
Default

I appreciate what CJT is saying which is why I included the
concept of boundary conditions. However to be more
technically correct, then most readers would not understand
the concepts. Yes, sine waves go forever in time. The 'sine
waves' that combine to construct a pulse have boundary
conditions; exist only during the period of that pulse.

Wavelet as a better description would be nice. But I
believe most don't even know what a wavelet is. I would not
even know how to begin to describe a lightning pulse in terms
of wavelets. Where would we even start? Define a pulse in
terms of a Daubechie, Mexican Hat, or Morlet wavelet? I
think not.

Furthermore, I was not sure of CJT's math background. For
example, what is a true impulse? Literally every frequency in
that spectrum. But did CJT understand that basic math
concept? I thought not after a few posts suggested I had
better keep it simpler: describe a pulse in terms that all may
understand - sine waves. Then made the concept a little more
complicated - added boundary conditions.

Wavelets - way too complex. To comprehend the destructive
(almost capricious) nature of lightning, it is simply better
to limit a lightning pulse description to a wide spectrum of
frequencies (numerous sine waves of different frequencies that
are summed together).

Leonard Caillouet wrote:
w_tom,

While you are correct in many ways, CJT is also correct in
pointing out that your suggestion that lightning is made of
sine waves is faulty application of Fourier. You could just as easily say that lightning is made of wavelets
(and more correctly, actually) or many other functions. These are all just
mathematical constructs to describe a complex phenomenon as components that
can be manipulated for analysis. The point that lightning is a pulse and
can be analyzed by its component frequencies should be clear. That it is
"made of sine waves" is an equally clearly faulty application of the
concept. If you would learn to be a bit more humble in accepting criticism
of your language, useage, and out of context application of concepts, the
correct basis of your arguments might be more often appreciated. Mostly,
you end up looking like an idiot.

Leonard

  #33   Report Post  
w_tom
 
Posts: n/a
Default

Pulses are sums of numerous frequencies - numerous sine
waves. A pulse is not just a pulse. Like all waveforms, the
pulse is also a sum of various sine waves (of different
frequency, amplitude, and phase). IOW a pulse is not just a
pulse. A pulse is what happens when numerous sine waves (with
boundary conditions) are summed together.

That is also true of lightning. Lightning is not some DC
pulse. Lightning is AC electricity - at numerous radio
frequencies.

I was trying to be clear. Lighting is electricity. But the
electricity called lightning does create electromagnetic waves
that will be of same frequency.

Ground wires from any surge protector must avoid sharp
bends. As I have so often done, a published source:
US Army Training Manual 5-690
3.4 Lightning protection subsystem (p 46)
d.1(i) Installation of surge arresters is shown for
grounded and ungrounded service ... In order to prevent
introducing excessive inductance and resistance in the
transient path to the surge arrester, No. 4 AWG (minimum)
insulated stranded copper wire of the minimum feasible
length must be used to make the interconnection(s) unless
otherwise recommended and guaranteed by the manufacturer.
Also, the interconnecting wiring must not contain loops
or sharp bends. Otherwise, the response time of the surge
arrester will be delayed and a higher clamp voltage than
that of the surge arrester will be impressed across the
protected equipment, thus increasing the possibility of
damage. In the event a very fast transient should occur,
it is quite likely that the surge arrester would never
turn on, and all of the transient energy would be
dissipated by supposedly protected equipment.


I never said the earthing wire for a lightning rod must not
be surrounded by a building's girders. I only said the
preferred method of routing that earthing wire is to keep it
outside the building. This for reasons beyond the scope of
this discussion which is impedance, the purpose of earthing,
and characteristics of a lightning pulse that makes it so
challenging.

CJT wrote:
Certainly pulses can excite tuned circuits. That's different
from what you have been saying.

A radio receives electromagnetic radiation at its tuned
frequency (assuming the signal is not so strong as to
overwhelm filters). If a pulse is only a pulse then the radio
does not receive anything. But since a pulse is AC
electricity at numerous frequencies, then the radio receives
only its unique frequency from that pulse.

I made it easy. Chart provides a frequency spectrum for a
lightning pulse. How can the pulse have a frequency spectrum
if a pulse is only a pulse? Yes some of those RF frequencies
are so low as to be same as audio frequencies. But they are
electrical - not mechanical motion. The electricity is still
RF frequencies. Some of the most destructive energy is found
in those higher (and less amplitude) frequencies.


You seem to be using "RF frequencies" to mean "electromagnetic
radiation." I think that's another example of imprecise language.

I guess you are finally coming to accept the fact that a
lightning pulse is not just a pulse (and not a DC pulse as
TimPerry claimed). You don't provide any supporting facts,
numbers, or citations for your claim that "a pulse is only a
pulse". Provided were numerous reasons why a lightning pulse
is composed of AC electricity at numerous RF frequencies.


I've deliberately steered clear of the DC vs AC controversy.
If pressed, I'd probably come down on the side of TimPerry,
because I find his flashlight analogy persuasive. But I don't
think the distinction is particularly useful, so I find it
uninteresting. You and TimPerry could probably resolve your
differences by careful definition of the terms DC and AC. I
suspect that under your definition, there has never been a true
DC (i.e. unvarying, as I understand your position) source.

Numerous sine waves summed together to create a pulse. To
repeat some supporting facts: We measure the frequency
spectrum of a pulse because a pulse is composed of so many
different frequencies. That lightning pulse is affected by
impedance because it contains radio frequencies - not DC. The
pulse contains frequencies that create noise on tuned radios.
Wire impedance created by sharp wire bends can undermine a
protection system because lightning is composed of AC
electricity.


Without some calculation to support it, that's a vacuous
statement. I think it's probably also incorrect in general.

You cited a surge arrestor company's Web site earlier. Do
you think the ground wires associated with surge arrestors
must similarly avoid sharp bends?

A chart with the frequency spectrum for a
lightning pulse is provided showing energy even at the
megahertz range. The so called DC arc is really AC
electricity. Even Fourier analysis demonstrates that all
waveforms (such as pulses) are summations of sine waves at
various amplitudes, frequencies, and phases.


At the risk of sounding too much like a former President, that
depends on your definition of "are." "Can be analyzed for
certain purposes as if they were" might be closer to the truth.

Lighting pulse
contains many sine waves (with boundary conditions). The
sharper that pulse, then the more frequencies are contained in
a pulse.

Meanwhile your concept of earthing a steel building is
flawed. The steel frame is sufficient to be an earth ground.
It does not have high impedance to obstruct a lightning
strike. However, for better protection, the lightning rod is
earthed using wire outside the building. Better protection
means a building's earthing meets lightning rod's earthing at
a point beneath the building - the single point grounding
concept. A building structure is not some big 'lightning
impeding' inductor. However even wire has impedance which is
why shorter connections to earth ground mean superior
lightning protection. Wire impedance is also why plug-in
protectors are not effectively earthed.


Apparently I was unclear. :-) I'm not aware of any requirement
that the wire grounding a lightning rod not be encircled at any
point by the building's girders. Another post of yours seemed
to imply such a requirement.

Lastly cited is how BT and other telcos earth their
switching stations to not suffer lightning damage. Every
incoming wire is earthed ideally 50 meters before those wires
connect to the computer. Connection from each incoming wire
to earth is as short as possible. A larger separation between
'earthing connection to computer' provides more impedance -
better computer protection. Short connection 'from incoming
wire to earth' means less impedance - a better path for
lightning. Why do we mention this? Because lightning
protection is about low impedance earthing. Better lightning
protection means the protector is where utility wires enter
the building, with a shortest connection to a single point
earth ground, and not adjacent to electronics.

A lightning pulse is not just a pulse. Lightning is many AC
waves in a wide frequency spectrum - AC electricity - that
seeks earth ground. Protection from lightning involves RF
principles that radio engineers best appreciate.

CJT wrote:
w_tom wrote:
The radio is tuned only to receive a specific frequency.
That pulse is composed of many frequencies.
The pulse is a pulse.

Frequency that
interferes with radio reception is a unique sine wave
frequency
within that pulse.
You're really hung up on those sine waves for some reason.
Fourier analysis is a mathematical construct (admittedly
a very useful one), but it's a fiction (in the best sense
of the word). The lightning isn't "created" from sine
waves.

Just one of so many frequencies that
create the pulse. Pulses - like all waveforms - are a
summation of basic sine waves at different frequencies,
amplitudes, and phases.
So what's the phase of the third sine wave in the series?
What physical phenomenon results in that phase? Does the
Fourier series you contend results in the single lightning
pulse match it through all time? Think about what you're
saying.

The radio does not receive a pulse.
Huh? Why not?

It receives only parts of a waveform that are specific to its
tuned frequency. Fourier analysis demonstrates the concept.
The radio does not receive a pulse so much as it receives one
frequency that was part of that pulse.
So what magic keeps the other parts of the pulse away from the radio?
Face it, the radio gets the full pulse and processes it into whatever
you hear.

A chart for the frequency spectrum of lightning is available
in:
http://www.hubersuhner.com/products/...kb-bas-fre.htm
.

Did you look at the scale on that chart? Is that your idea of RF?
Looks more like audio to me.

What does DC arcing create? That was how radios worked.
Sparking DC electricity created AC electricity that resulted
in radio waves. Telsa did not transmit DC electricity. To
perform electricity transmissions, DC electricity was
converted to AC. DC arcing is detected how? By measuring AC
components created by that arcing.
You're arguing with yourself. I didn't say it was DC (or AC for
that matter -- the DC/AC distinction, as explored recently in
another thread, is not necessarily useful in a discussion such
as this).

Meanwhile TimPerry repeatedly refers to DC pulses. That
would be DC analysis. Lightning pulse is AC electricity. It
creates electromagnetic waves of same frequencies. You even
have a chart for that frequency spectrum. What the chart does
not show is how the energy content quickly tapers to zero as
frequency approaches DC. That pulse called lightning is
composed of electricity at RF frequencies.

I suspect we're more in agreement than would appear. But my problem
with your position is twofold:

- your insistence that a pulse in nature is somehow created from lots
of sine waves; maybe it's just the words you use, but I have a
philosophical problem with projecting a mathematical analysis
technique onto nature as more than an analogy

- (not explored here yet, but related) your claim in one of your
posts that a sharp bend in a ground wire would necessarily destroy
its effectiveness in shunting lightning to ground because of effects
on its impedance. I think you also contend that wrapping a coil
around the wire would cause a similar problem. I assume you would
then contend that all skyscrapers run their lightning rod ground wires
outside their girder structures, to avoid being encompassed by single
turn coils consisting of metallic girders.


--
The e-mail address in our reply-to line is reversed in an attempt to
minimize spam. Our true address is of the form .

  #34   Report Post  
CJT
 
Posts: n/a
Default

w_tom wrote:
snip

At any rate, perspective of this discussion is Lightning
protection. (Although it originally was about a faraday cage
provided by a chain link cage), it has morphed into the AC
nature of lightning; the component that makes lightning so
challenging and destructive. So challenging that some call
lightning capricious. The AC component in lightning that make
protection difficult is not intuitively obvious. The
components that tend to be most challenging are in the
kilohertz and megahertz range. IOW the DC component is rarely
considered in the overall design of lightning protection.
Higher frequency components are discussed as if higher
frequencies are the only part that exists.

snip

Think about what you just wrote. If you size the ground wire to
a lightning rod only on the basis of high frequency components,
and don't account for the essentially DC component (which can
exceed 10,000 amps, according to the Web site you cited earlier),
you won't have much protection.

--
The e-mail address in our reply-to line is reversed in an attempt to
minimize spam. Our true address is of the form .
  #35   Report Post  
TimPerry
 
Posts: n/a
Default


"Asimov" wrote in message
...
"w_tom" bravely wrote to "All" (07 Jul 05 22:29:12)
--- on the heady topic of " Lightning protection"

w_ From: w_tom
w_ Xref: aeinews sci.electronics.repair:52721

w_ Meanwhile TimPerry repeatedly refers to DC pulses. That
w_ would be DC analysis. Lightning pulse is AC electricity. It
w_ creates electromagnetic waves of same frequencies. You even
w_ have a chart for that frequency spectrum. What the chart does
w_ not show is how the energy content quickly tapers to zero as
w_ frequency approaches DC. That pulse called lightning is
w_ composed of electricity at RF frequencies.

He is not completely wrong, Tom.


alas: this implys i'm not completely right .


There is a phenomenon of charge
separation which takes place in the cloud and there is a DC potential.
The initial discharge starts with DC but with distance and motion we
have the effects of inductance and capacitance which establish an
impedance and results in reflections where discontinuities exist for a
moving wavefront.


What we basically have is a square wave going from
one DC potential to ground but, like the Fourrier model, made up of an
infinite series of sine waves. So yes, there will be AC


but only as a theoretical concept. the electrons only travel on one
direction. cloud to earth or earth to cloud, or cloud to cloud. (mostly)



but the
initial natural process is all about charge separation in the clouds.
It can be no other way.

A*s*i*m*o*v

... Power is obtained by current meeting resistance



This week a master electrician asked me how well versed I was in lightning.
I looked him straight in the eye and said "there is no human on the planet
that's well versed in lightning". He cracked up and nearly fell down
laughing.

He then gasped out "that's exactly what I wanted to hear"

We then went on to discuss his problem. he had installed some motors with
solid state starter circuits in a grain elevator 2 years ago. this year a
series of storms has blown the starter/controller on multiple occasions. The
frustrated granary owner accused master electrician Frank of installing
faulty grounds.

I explained that from my viewpoint that the initial causation of equipment
damage can be mighty hard determine. Did the lightning come sown the
structure? Was it a hit on the power line or transformer? was is a spike
caused by brief power interruption? in many cases I am faced with blown
fuses, open breakers, transformers that are shorted and sometimes on fire,
and sometimes arc holes in equipment cabinets. At this time I am replacing
some arc gaps and lighting transformers that were melted by a stroke. The
project will cost about $15,000 USD as some aircraft beacons will need to be
replaced along with AC cable.

We then proceeded to plan the install of a 100 kW generator. I pointed out
the parallel solid state TVSS unit and the 4" ferrite toroids on the load
side of the 3 phase service. He said "wow does that prevent lightning
damage?" I replied when is comes to direct hits by lightning anything can
happen but over the long term this and similar arrangements reduce
maintenance / repair costs and downtime.

for those interested in reading a "white paper" on lightning protection this
page http://www.nautel.com/support/files/...protection.pdf may be
informative.

It is geared toward a specific type of situation. it lists sources.

section 2.2 describes lightning characteristics. you may agree or disagree
with the data it presents but you may note

section 2.2.2 describes a median main lightning strike pulse as a
uni-directional near-exponential pulse of 20,000 amperes peak amplitude
lasting 40 microseconds to half amplitude.

I don't know about you but uni-directional pulse sounds like DC to me.








  #36   Report Post  
w_tom
 
Posts: n/a
Default

10,000 amp for what time? Some assume a 12 AWG wire (2mm)
used for 20 amp service will vaporize under higher currents.
First that same wire will carry 200 amps continuous without
vaporizing. Second, did you notice the word 'continuos'. Now
lets limit that current to 40 microseconds. That wire can
easily carry 10,000 amps for a short time span. Easily?
Well, complications are created when the wire has sharp bends,
loops, routed inside metallic conduit, etc.

I can appreciate your apprehension. To you, much of this is
new. But the technology is old, standard, and well proven for
decades. Earthing wires required for AC electric must be so
large that the DC component in lightning is irrelevant.
Complications are created when that wire is not properly
installed so that wire impedance is increased.

What makes lightning protection even more challenging is
that humans don't learn of their mistakes until after the
first lightning strike. Humans made a mistake that only humans
can correct.

CJT wrote:
Think about what you just wrote. If you size the ground wire to
a lightning rod only on the basis of high frequency components,
and don't account for the essentially DC component (which can
exceed 10,000 amps, according to the Web site you cited earlier),
you won't have much protection.

  #37   Report Post  
TimPerry
 
Posts: n/a
Default

Ironic that TimPerry would say DC triangle waves are not
confusing


since that is not what he said on 11 Jun 2005 in the
newsgroup sci.electronics.basics entitled "DC Wave
Questions":
There are a number of waveforms that go in only one
direction relative to ground such as sawtooth waves, square
waves, and triangle waves. To refer to these as say a "DC
triangle wave" would be equally confusing without further
qualification.


an equally long and ultimately fruitless discussion initiated by a student
who used the term DC sinewave.

please note the wording. neither statemet advocates the use of the phrase
"DC wave" "or DC triangle wave"

"nothing confusing about it to me. if a waveform lies above reference ground
it becomes fluctuating DC regardless of its shape, frequency, pulse
repetition rate. id just call it a triangle wave or sawtooth or whatever.
the DC part is usully understood."

Tom, my objection was to your repeated declaration that "the term DC pulse
is an oxymoron". however my statement comes off in a way I did not intend.
Please accept my apology. I agree with you that the term DC triangle wave is
confusing.


Meanwhile, concrete is not an insulator as TimPerry would
have us believe. Concrete is a conductor.


yee haw we wont need to bother with expensive copper anymore. Just pour us
some concrete power lines.


Concrete is why
Ufer grounds are a most effective method of protecting
transistors ...


i should wrap all my transistors in concrete?

or protecting munitions (the original purpose
of Ufer grounds).

and the steel rebar has nothing to do with it?





  #38   Report Post  
CJT
 
Posts: n/a
Default

w_tom wrote:
10,000 amp for what time? Some assume a 12 AWG wire (2mm)
used for 20 amp service will vaporize under higher currents.
First that same wire will carry 200 amps continuous without
vaporizing. Second, did you notice the word 'continuos'. Now
lets limit that current to 40 microseconds. That wire can
easily carry 10,000 amps for a short time span. Easily?
Well, complications are created when the wire has sharp bends,
loops, routed inside metallic conduit, etc.

I can appreciate your apprehension. To you, much of this is
new. But the technology is old, standard, and well proven for
decades. Earthing wires required for AC electric must be so
large that the DC component in lightning is irrelevant.
Complications are created when that wire is not properly
installed so that wire impedance is increased.

What makes lightning protection even more challenging is
that humans don't learn of their mistakes until after the
first lightning strike. Humans made a mistake that only humans
can correct.

Your patronizing attitude is starting to get annoying.

Your argument can be extended to the "AC" component on which you
focus -- if it's all over in 40 microseconds, what's the big deal?

:-)

CJT wrote:
Think about what you just wrote. If you size the ground wire to
a lightning rod only on the basis of high frequency components,
and don't account for the essentially DC component (which can
exceed 10,000 amps, according to the Web site you cited earlier),
you won't have much protection.



--
The e-mail address in our reply-to line is reversed in an attempt to
minimize spam. Our true address is of the form .
  #39   Report Post  
Choreboy
 
Posts: n/a
Default

w_tom wrote:


That is also true of lightning. Lightning is not some DC
pulse. Lightning is AC electricity - at numerous radio
frequencies.


Isn't the definition of AC electricity whose current changes direction?
Are you saying all lightning strikes change direction?


Ground wires from any surge protector must avoid sharp
bends. As I have so often done, a published source:
US Army Training Manual 5-690
3.4 Lightning protection subsystem (p 46)
d.1(i) Installation of surge arresters is shown for
grounded and ungrounded service ... In order to prevent
introducing excessive inductance and resistance in the
transient path to the surge arrester, No. 4 AWG (minimum)
insulated stranded copper wire of the minimum feasible
length must be used to make the interconnection(s) unless
otherwise recommended and guaranteed by the manufacturer.
Also, the interconnecting wiring must not contain loops
or sharp bends. Otherwise, the response time of the surge
arrester will be delayed and a higher clamp voltage than
that of the surge arrester will be impressed across the
protected equipment, thus increasing the possibility of
damage. In the event a very fast transient should occur,
it is quite likely that the surge arrester would never
turn on, and all of the transient energy would be
dissipated by supposedly protected equipment.


I wonder why they specify insulated wire. Uninsulated wire is easier to
inspect and easier to identify as a ground. Stranded wire has less
inductance than solid, but what about flat braid? I think it's
specified for commercial transmission towers because it has less
inductance than stranded wire.

If an Army technician had thirty feet of ground wire to connect
terminals ten feet apart, he might leave the extra twenty feet taped in
a coil halfway between. I agree that a loop like that would be bad. By
comparison, how bad would it be to have a sharp bend around a wooden corner?

I never said the earthing wire for a lightning rod must not
be surrounded by a building's girders. I only said the
preferred method of routing that earthing wire is to keep it
outside the building. This for reasons beyond the scope of
this discussion which is impedance, the purpose of earthing,
and characteristics of a lightning pulse that makes it so
challenging.

An external ground wire would be easy to inspect and less likely to be
damaged by humans. Are there more technical reasons?
  #40   Report Post  
Choreboy
 
Posts: n/a
Default

w_tom wrote:

The term DC pulses is an oxymoron.


Isn't DC electrical current that flows in only one direction? In 1883,
when a telegraph operator hooked up a battery and started pressing his
key, wasn't he generating DC pulses? (There may have been a little AC
activity each time the key broke contact.)

At any rate, perspective of this discussion is Lightning
protection. (Although it originally was about a faraday cage
provided by a chain link cage), it has morphed into the AC
nature of lightning; the component that makes lightning so
challenging and destructive. So challenging that some call
lightning capricious. The AC component in lightning that make
protection difficult is not intuitively obvious. The
components that tend to be most challenging are in the
kilohertz and megahertz range. IOW the DC component is rarely
considered in the overall design of lightning protection.
Higher frequency components are discussed as if higher
frequencies are the only part that exists.


I don't know if they were familiar with AC when they defined the Henry.
Across one Henry of inductance, it takes one Volt to increase the
electrical flow by one Ampere per second.

Suppose a grounded terminal gets hit by a 1000-Volt pulse lasting .001
second. Suppose the grounding rod has 20 ohms to ground, and there's a
1 Henry choke between the terminal and the rod.

The thousand Volts will increase the current through the inductor at
1000 Amperes per second. At the end of 1 millisecond, current will have
increased to one Amp. In view of the resistance at the ground rod,
voltage will have increased to 20 Volts.

Things will be entirely different on the hot side of the choke. What
appears to be a short for unvarying DC will appear to be an open for a
quick, low-impedance pulse. Who needs AC theory for that?
Reply
Thread Tools Search this Thread
Search this Thread:

Advanced Search
Display Modes

Posting Rules

Smilies are On
[IMG] code is On
HTML code is Off
Trackbacks are On
Pingbacks are On
Refbacks are On


Similar Threads
Thread Thread Starter Forum Replies Last Post
Lightning Protection Ross Mac Home Repair 42 March 4th 05 05:41 AM
OT Guns more Guns Cliff Metalworking 519 December 12th 04 05:52 AM
Thermal Protection Rating on small electric motors 1_Patriotic_Guy Home Repair 6 October 18th 04 11:48 PM
Are PC surge protectors needed in the UK? greywolf42 Electronics Repair 82 July 13th 04 11:37 AM
Difference between whole-house surge supressor and secondary surge arrestor Vinnie Murdico Home Repair 4 September 2nd 03 12:52 PM


All times are GMT +1. The time now is 10:05 PM.

Powered by vBulletin® Copyright ©2000 - 2024, Jelsoft Enterprises Ltd.
Copyright ©2004-2024 DIYbanter.
The comments are property of their posters.
 

About Us

"It's about DIY & home improvement"